5
大学数学基礎解説
文献あり

積分botを解けるだけ解く, その3

361
0
$$\newcommand{erfc}[0]{\mathrm{erfc}} $$

前回( https://mathlog.info/articles/1901 )

21個目

$$\begin{eqnarray*} \int_0^{\pi/2}\arctan\left(1-\frac{\sin^2 2x}{4}\right)\,dx=\frac{\pi^2}{4}-\pi\arctan\sqrt{\frac{\sqrt{2}-1}{2}} \end{eqnarray*}$$

これはなかなか計算が大変でした.

まず, $0\lt a$に対し,
$$\begin{eqnarray*} \int_0^{\pi/2}\frac 1{a^2+\sin^2 x}\,dx&=&\left[\frac{1}{a\sqrt{1+a^2}}\arctan\left(\frac{\sqrt{1+a^2}}{a}\tan x\right)\right]_0^{\pi/2}\\ &=&\frac{\pi}{2a\sqrt{1+a^2}} \end{eqnarray*}$$
よって, 解析接続により, $0\lt Re(a)$に対し,
$$\begin{eqnarray*} \int_0^{\pi/2}\ln(a^2+\sin^2 x)\,dx=\pi\sinh^{-1}a+C \end{eqnarray*}$$
となる定数$C$がある. $a=0$として, $C=-\pi\ln 2$だから,
$$\begin{eqnarray*} \int_0^{\pi/2}\ln(a^2+\sin^2 x)\,dx=\pi(\sinh^{-1}a-\ln 2) \end{eqnarray*}$$
よって,
$$\begin{eqnarray*} \int_0^{\pi/2}\arctan\left(1-\frac{\sin^2 2x}{4}\right)\,dx&=&\Im\left(\int_0^{\pi/2}\ln(4+i(4-\sin^2 x))\,dx\right)\\ &=&\Im\left(\int_0^{\pi/2}\ln(4(1+i)-i\sin^2 x))\,dx\right)\\ &=&\Im\left(\int_0^{\pi/2}\left(\ln(4(i-1)+\sin^2 x))-\frac{\pi i}{2}\right)\,dx\right)\\ &=&-\frac{\pi^2}{4}+\Im\left(\int_0^{\pi/2}\ln((2^{5/4}e^{3i\pi/8})^2+\sin^2 x)\,dx\right)\\ &=&-\frac{\pi^2}{4}+\pi\Im(\sinh^{-1}(2^{5/4}e^{3i\pi/8})) \end{eqnarray*}$$
ここで,
$$\begin{eqnarray*} 2^{5/4}e^{3i\pi/8}=\sqrt{2(\sqrt{2}-1)}+i\sqrt{2(\sqrt{2}+1)} \end{eqnarray*}$$
より,
$$\begin{eqnarray*} \sinh^{-1}(2^{5/4}e^{3i\pi/8})&=&\ln\left(\sqrt{2(\sqrt{2}-1)}+i\sqrt{2(\sqrt{2}+1)}+\sqrt{1+\left(\sqrt{2(\sqrt{2}-1)}+i\sqrt{2(\sqrt{2}+1)}\right)^2}\right)\\ &=&\ln\left(\sqrt{2(\sqrt{2}-1)}+i\sqrt{2(\sqrt{2}+1)}+\sqrt{4i-3}\right)\\ &=&\ln\left(1+\sqrt{2(\sqrt{2}-1)}+i\left(2+\sqrt{2(\sqrt{2}+1)}\right)\right)\\ \end{eqnarray*}$$
よって,
$$\begin{eqnarray*} \Im(\sinh^{-1}(2^{5/4}e^{3i\pi/8}))&=&\arctan\frac{2+\sqrt{2(\sqrt{2}+1)}}{1+\sqrt{2(\sqrt{2}-1)}}\\ &=&\arctan\sqrt{2(\sqrt{2}+1)}\\ &=&\frac{\pi}2-\arctan\sqrt{\frac{2}{\sqrt{2}-1}} \end{eqnarray*}$$
これを先ほどの式に代入すればよい.

22個目( https://twitter.com/integralsbot/status/1361264323922780163 )

$$\begin{eqnarray*} \int_0^{\infty}\frac{\arctan\sqrt{a^2+x^2}}{(1+x^2)\sqrt{a^2+x^2}}\,dx=\frac{\pi}{\sqrt{a^2-1}}\arctan\frac{\sqrt{a^2-1}}{2+\sqrt{a^2+1}} \end{eqnarray*}$$

引き続き計算が重そうな式ですね. Fubiniの定理を使ってみましょう.

Fubiniの定理より,
$$\begin{eqnarray*} &&\int_0^{\infty}\frac{\arctan\sqrt{a^2+x^2}}{(1+x^2)\sqrt{a^2+x^2}}\,dx\\ &=&\int_0^{\infty}\int_0^1\frac{1}{(1+x^2)(1+(a^2+x^2)y^2)}\,dydx\\ &=&\int_0^1\int_0^{\infty}\frac{1}{(1+x^2)(1+a^2y^2+x^2y^2)}\,dxdy\\ &=&\frac{\pi}{2}\int_0^1\frac{1}{\sqrt{1+a^2y^2}(y+\sqrt{1+a^2y^2})}dy\\ &=&\frac{\pi}2\int_0^{\sinh^{-1}a}\frac{1}{\sinh x+a\cosh x}\,dx\\ &=&\frac{\pi}{2}\left[\frac{1}{\sqrt{a^2-1}}\arctan\left(\sinh\left(x+\tanh^{-1}\frac 1a\right)\right)\right]_0^{\sinh^{-1}a}\\ &=&\frac{\pi}{2\sqrt{a^2-1}}\left(\arctan\left(\sinh\left(\sinh^{-1}a+\tanh^{-1}\frac 1a\right)\right)-\arctan\left(\sinh\left(\tanh^{-1}\frac 1a\right)\right)\right)\\ &=&\frac{\pi}{2\sqrt{a^2-1}}\left(\arctan\left(\frac{a^2+\sqrt{a^2+1}}{\sqrt{a^2-1}}\right)-\arctan\left(\frac{1}{\sqrt{a^2-1}}\right)\right)\\ &=&\frac{\pi}{2\sqrt{a^2-1}}\arctan\left(\frac{(a^2-1+\sqrt{a^2+1})\sqrt{a^2-1}}{2a^2-1+\sqrt{a^2+1}}\right)\\ \end{eqnarray*}$$
ここから$\arctan$を変形していきます.
$$\begin{eqnarray*} &&\arctan\left(\frac{(a^2-1+\sqrt{a^2+1})\sqrt{a^2-1}}{2a^2-1+\sqrt{a^2+1}}\right)\\ &=&\arctan\left(\frac{(\sqrt{a^2+1}-1)(\sqrt{a^2+1}+2)\sqrt{a^2-1}}{(\sqrt{a^2+1}-1)(2\sqrt{a^2+1}+3)}\right)\\ &=&\arctan\left(\frac{(\sqrt{a^2+1}+2)\sqrt{a^2-1}}{2\sqrt{a^2+1}+3}\right)\\ &=&\arctan\left(\frac{2\sqrt{a^2-1}(2+\sqrt{a^2+1})}{(2+\sqrt{a^2+1})^2-a^2+1}\right))\\ &=&\arctan\left(\frac{\frac{2\sqrt{a^2-1}}{2+\sqrt{a^2+1}}}{1-\left(\frac{\sqrt{a^2-1}}{2+\sqrt{a^2+1}}\right)^2}\right)\\ &=&2\arctan\frac{\sqrt{a^2-1}}{2+\sqrt{a^2+1}} \end{eqnarray*}$$
これを先ほどの式に代入すればよい.

23個目( https://twitter.com/integralsbot/status/1361269865793921028 )

$$\begin{eqnarray*} \int_0^1\frac{\ln x\arcsin xy}{x(1-x^2)}\,dx=\frac 16\arcsin^3 y-\frac 34\zeta(2)\arcsin y \end{eqnarray*}$$

とりあえず, 項別積分でうまくいきそうですね.

まず, 最初に
$$\begin{eqnarray*} \arcsin x&=&\sum_{0\leq n}\frac{\binom{2n}{n}}{2^{2n}(2n+1)}x^{2n+1}\\ \frac 16\arcsin^3 x&=&\sum_{0\leq n}\frac{\binom{2n}{n}}{2^{2n}(2n+1)}x^{2n+1}\sum_{m=0}^{n-1}\frac{1}{(2m+1)^2} \end{eqnarray*}$$
であることを思い出しておきます. (証明は https://www.researchgate.net/publication/26532996_Integer_Powers_of_Arcsin を参照)
$$\begin{eqnarray*} \int_0^1\frac{\ln x\arcsin xy}{x(1-x^2)}\,dx&=&\sum_{0\leq n}\frac{\binom{2n}{n}}{2^{2n}(2n+1)}y^{2n+1}\int_0^1\frac{x^{2n+1}\ln x}{1-x^2}\,dx\\ &=&\sum_{0\leq n}\frac{\binom{2n}{n}}{2^{2n}(2n+1)}y^{2n+1}\sum_{n\leq m}\int_0^1x^{2m+1}\ln x\,dx\\ &=&-\sum_{0\leq n}\frac{\binom{2n}{n}}{2^{2n}(2n+1)}y^{2n+1}\sum_{n\leq m}\frac{1}{(2m+1)^2}\\ &=&\sum_{0\leq n}\frac{\binom{2n}{n}}{2^{2n}(2n+1)}y^{2m+1}\sum_{m=0}^{n-1}\frac{1}{(2m+1)^2}-\sum_{m=0}^{\infty}\frac 1{(2m+1)^2}\sum_{0\leq n}\frac{\binom{2n}{n}}{2^{2n}(2n+1)}y^{2n+1}\\ &=&\frac 16\arcsin^3 y-\frac 34\zeta(2)\arcsin y \end{eqnarray*}$$

24個目( https://twitter.com/integralsbot/status/1361280330779426818 )

$$\begin{eqnarray*} \int_{-\infty}^{\infty}\left(\frac{x}{2+e^x+e^{-x}}\right)^2\,dx=\frac{\zeta(2)-1}{3} \end{eqnarray*}$$

とりあえず, Mellin変換すればいい感じですね.

$$\begin{eqnarray*} \int_{-\infty}^{\infty}\left(\frac{x}{2+e^x+e^{-x}}\right)^2\,dx&=&2\int_{0}^{\infty}\left(\frac{x}{(e^{x/2}+e^{-x/2})^2}\right)^2\,dx\\ &=&16\int_0^{\infty}\frac{x^2}{(e^x+e^{-x})^4}\,dx \end{eqnarray*}$$
ここで, $3\lt Re(s)$として, Mellin変換を考える.
$$\begin{eqnarray*} \int_0^{\infty}\frac{x^{s-1}}{(e^x+e^{-x})^4}\,dx&=&\frac 16\sum_{1\lt n}(-1)^{n}(n-1)n(n+1)\int_0^{\infty}x^{s-1}e^{-2nx}\,dx\\ &=&\frac {2^{-s}}6\sum_{1\lt n}(-1)^n\frac{n^2-1}{n^{s-1}}\\ &=&\frac {2^{-1-s}}3\sum_{0\lt n}(-1)^{n-1}\left(\frac 1{n^{s-1}}-\frac{1}{n^{s-3}}\right)\\ &=&\frac {2^{-1-s}}{3}(\eta(s-1)-\eta(s-3)) \end{eqnarray*}$$
ここで, $\eta(s)$はDirichletのイータ関数である. $s\to 3$として,
$$\begin{eqnarray*} \int_0^{\infty}\frac{x^2}{(e^x+e^{-x})^4}\,dx&=&\lim_{s\to 3}\frac{2^{-1-s}}{3}(\eta(s-1)-\eta(s-3))\\ &=&\frac{1}{48}(\zeta(2)-1) \end{eqnarray*}$$
これを先ほどの式に代入すればよい.

25個目( https://twitter.com/integralsbot/status/1361287489961525251 )

$$\begin{eqnarray*} \int_0^1\frac{\sin(n\sin^{-1}x)\sinh\left(n\sinh^{-1}\frac xa\right)}{\cos(2n\sin^{-1}x)+\cosh\left(2n\sinh^{-1}\frac xa\right)}\frac{dx}{x\sqrt{1-x^2}\sqrt{1+\frac{x^2}{a^2}}}=\frac{\tan^{-1}a}{2}\quad(n:odd) \end{eqnarray*}$$

これは, 全く分からない. 被積分関数にいい感じの性質があるんですかね.

26個目( https://twitter.com/integralsbot/status/1361301353319059457 )

$$\begin{eqnarray*} \int_{-\infty}^{\infty}\frac{\sin\left(x-\frac ax\right)}{x+\frac 1x}\,dx=\frac{\pi}{e^{1+a}} \end{eqnarray*}$$

留数定理か, と思ったけど, なんか$x=0$に真性特異点あるので, 微分方程式でやってみます.

まず, $x\to\infty$$\sin\left(x+\frac ax\right)\to \sin x$で, $x\to\frac 1x$の置換により, この積分の収束性が分かる. $0\lt a$に対し,
$$\begin{eqnarray*} I(a):=\int_{-\infty}^{\infty}\frac{x\sin\left(x-\frac ax\right)}{1+x^2}\,dx \end{eqnarray*}$$
とする.
$$\begin{eqnarray*} I''(a):=-\int_{-\infty}^{\infty}\frac{\sin\left(x-\frac ax\right)}{x(1+x^2)}\,dx \end{eqnarray*}$$
より,
$$\begin{eqnarray*} I(a)-I''(a)&=&\int_{-\infty}^{\infty}\frac{\sin\left(x-\frac ax\right)}{x}\,dx\\ &=&\int_{-\infty}^{\infty}\frac{\sin\left(\frac ax-x\right)}{\frac ax}\frac{a}{x^2}\,dx\\ &=&-\int_{-\infty}^{\infty}\frac{\sin\left(x-\frac ax\right)}{x}\,dx\\ &=&0 \end{eqnarray*}$$
よって, ある定数$C_1,C_2$が存在して,
$$\begin{eqnarray*} I(a)=C_1e^a+C_2e^{-a} \end{eqnarray*}$$
よく知られた等式,
$$\begin{eqnarray*} \int_{-\infty}^{\infty}\frac{\cos ax}{1+x^2}\,dx&=&\pi e^{-a}\\ \int_{-\infty}^{\infty}\frac{x\sin ax}{1+x^2}\,dx&=&\pi e^{-a} \end{eqnarray*}$$
より, $I(0)=\frac{\pi}{e}, I'(0)=-\frac{\pi}{e}$より, $C_1=0, C_2=\frac{\pi}{e}$だから,
$$\begin{eqnarray*} I(a)=\frac{\pi}{e^{1+a}} \end{eqnarray*}$$

27個目( https://twitter.com/integralsbot/status/1361316409138159616 )

$$\begin{eqnarray*} \int_{-\infty}^{\infty}\frac{dx}{(e^x+x+1)^2+\pi^2}=\frac 23 \end{eqnarray*}$$

今度こそ留数定理ですね.

まず, 方程式$e^x=1+x$の解を考えます. それはLambertの$W$関数を用いて,
$$\begin{eqnarray*} e^x&=&1+x\\ -(1+x)e^{-1-x}&=&-\frac 1{e}\\ -1-x&=&W_n\left(-\frac 1e\right)\\ x&=&-W_n\left(-\frac 1e\right)-1 \end{eqnarray*}$$
ここで, 虚部の絶対値が$\pi$以下であるものが$-W_0\left(-\frac 1e\right)-1=0$だけであることをグラフなどで確認しておきます.
積分路1 積分路1
ここで, 上のような積分路$C$を考えます. 留数定理より,
$$\begin{eqnarray*} Res_{x=0}\frac{1}{1+x-e^x}&=&\frac 1{2\pi i}\left(\int_{-R-\pi i}^{R-\pi i}-\int_{-R+\pi i}^{R+\pi i}\right)\frac 1{1+x-e^x}\,dx+\frac 1{2\pi i}\left(\int_{R-\pi i}^{R+\pi i}-\int_{-R-\pi i}^{-R+\pi i}\right)\frac 1{1+x-e^x}\,dx\\ &=&\frac 1{2\pi i}\int_{-R}^R\left(\frac 1{1+x+e^x-\pi i}-\frac 1{1+x+e^x+\pi i}\right)\,dx+\frac 1{2\pi i}\left(\int_{R-\pi i}^{R+\pi i}-\int_{-R-\pi i}^{-R+\pi i}\right)\frac 1{1+x-e^x}\,dx\\ &=&\int_{-R}^R\frac{1}{(e^x+x+1)^2+\pi^2}\,dx+\frac 1{2\pi i}\left(\int_{R-\pi i}^{R+\pi i}-\int_{-R-\pi i}^{-R+\pi i}\right)\frac 1{1+x-e^x}\,dx \end{eqnarray*}$$
ここで, $R\to\infty$として, 第2項が$0$に収束することにより,
$$\begin{eqnarray*} \int_{-\infty}^{\infty}\frac{1}{(e^x+x+1)^2+\pi^2}\,dx&=&Res_{x=0}\frac 1{1+x-e^x} \end{eqnarray*}$$
最後に留数を求めて,
$$\begin{eqnarray*} Res_{x=0}\frac 1{1+x-e^x}=\frac 23 \end{eqnarray*}$$

28個目( https://twitter.com/integralsbot/status/1361325388425699329 )

$$\begin{eqnarray*} \int_0^{\infty}\sqrt{\frac{\sqrt{\alpha^2+x^2}-\alpha}{\alpha^2+x^2}}\frac{\sin\gamma x}{e^{\beta\sqrt{\alpha^2+x^2}}}\,dx=\sqrt{\frac{\pi}{2}}\frac{\gamma e^{-\alpha\sqrt{\beta^2+\gamma^2}}}{\sqrt{\beta^2+\gamma^2}\sqrt{\beta+\sqrt{\beta^2+\gamma^2}}} \end{eqnarray*}$$

見た目がすごいですね, ちょっと形を整理してみますと,
$$\begin{eqnarray*} \int_0^{\infty}\sqrt{\frac{\sqrt{a^2+x^2}-a}{a^2+x^2}}e^{-b\sqrt{a^2+x^2}}\sin cx\,dx=\sqrt{\frac{\pi}{2}}\sqrt{\frac{\sqrt{b^2+c^2}-b}{b^2+c^2}}e^{-a\sqrt{b^2+c^2}} \end{eqnarray*}$$

$a=0$の場合はMellin変換
$$\begin{eqnarray*} \int_0^{\infty}x^{s-1}e^{-bx}\sin cx\,dx=\Gamma(s)(b^2+c^2)^{-s/2}\sin\left(s\arctan\frac cb\right) \end{eqnarray*}$$
$s=\frac 12$とした場合になっています. ここで,
$$\begin{eqnarray*} \sqrt{\frac{\sqrt{a^2+x^2}-a}{a^2+x^2}}=\sqrt{2}\Im\left(\frac 1{\sqrt{a-ix}}\right) \end{eqnarray*}$$
と書き直せることに気づきました. が, 解けませんでいた.

29個目( https://twitter.com/integralsbot/status/1361330254808236035 )

$$\begin{eqnarray*} \int_0^1\frac{x^{\mu+1/2}(1-x)^{\mu-1/2}}{(c+2bx-ax^2)^{\mu+1}}\,dx=\frac{\sqrt{\pi}}{\left(a+(\sqrt{c+2b-a}+\sqrt{c})^2\right)^{\mu+1/2}\sqrt{c+2b-a}}\frac{\Gamma\left(\mu+\frac 12\right)}{\Gamma(\mu+1)} \end{eqnarray*}$$

めっちゃ難しかった, うまい置換が思いつくまで, 全然分からない. でもやっぱ超幾何関数好きとしては, こういうのはちゃんと証明しておきたいと思ってます.

とりあえず, $2b$$b$に, $a$$-a$に, $\mu$$s$に置き換えて,
$$\begin{eqnarray*} \int_0^1\frac{x^{s+1/2}(1-x)^{s-1/2}}{(ax^2+bx+c)^{s+1}}\,dx=\frac{\sqrt{\pi}}{\left((\sqrt{a+b+c}+\sqrt{c})^2-a\right)^{s+1/2}\sqrt{a+b+c}}\frac{\Gamma\left(s+\frac 12\right)}{\Gamma(s+1)} \end{eqnarray*}$$
を示します.

まず, 両辺の連続性から, $a\neq 0$として証明すれば十分である.
$$\begin{eqnarray*} \int_0^1\frac{x^{s+1/2}(1-x)^{s-1/2}}{(ax^2+bx+c)^{s+1}}\,dx&=&a^{-1-s}\int_0^1x^{s+1/2}(1-x)^{s-1/2}\left(x^2+\frac ba x+\frac ca\right)^{-s-1}\,dx \end{eqnarray*}$$

なので, 最初から$a=1$としてよい. $x^2+bx+c=(1-(1-\alpha)x)(1-(1-\beta)x)$と因数分解すると(解析接続により, $\alpha,\beta$は正の実数と仮定して示せば十分である), 右辺は,
$$\begin{eqnarray*} &&\frac{\sqrt{\pi}}{\left((1+\sqrt{\alpha\beta})^2-(1-\alpha)(1-\beta)\right)^2\sqrt{\alpha\beta}}\frac{\Gamma\left(s+\frac 12\right)}{\Gamma(s+1)}\\ &=&\frac{\sqrt{\pi}}{\left((\sqrt{\alpha}+\sqrt{\beta})^2\right)^{s+1/2}\sqrt{\alpha\beta}}\frac{\Gamma\left(s+\frac 12\right)}{\Gamma(s+1)}\\ &=&\frac{\sqrt{\pi}}{(\sqrt{\alpha}+\sqrt{\beta})^{2s+1}\sqrt{\alpha\beta}}\frac{\Gamma\left(s+\frac 12\right)}{\Gamma(s+1)}\\ \end{eqnarray*}$$
よって,
$$\begin{eqnarray*} \int_0^1\frac{x^{s+1/2}(1-x)^{s-1/2}}{((1-(1-\alpha)x)(1-(1-\beta)x))^{s+1}}\,dx=\frac{\sqrt{\pi}}{(\sqrt{\alpha}+\sqrt{\beta})^{2s+1}\sqrt{\alpha\beta}}\frac{\Gamma\left(s+\frac 12\right)}{\Gamma(s+1)} \end{eqnarray*}$$
を示せばよい. $x\to\frac{x}{1+x}$の置換により,
$$\begin{eqnarray*} &&\int_0^1\frac{x^{s+1/2}(1-x)^{s-1/2}}{((1-(1-\alpha)x)(1-(1-\beta)x))^{s+1}}\,dx\\ &=&\int_0^{\infty}\frac{x^{s+1/2}}{((1+\alpha x)(1+\beta x))^{s+1}}\,dx\\ &=&\frac {1}{\alpha^{s+3/2}}\int_0^{\infty}\frac{x^{s+1/2}}{\left((1+x)\left(1+\frac{\beta}{\alpha}x\right)\right)^{s+1}}\,dx \end{eqnarray*}$$
より, 最初から$\alpha=1$としてよい, よって,
$$\begin{eqnarray*} \int_0^1x^{s+1/2}(1-x)^{s-1/2}(1-(1-\beta)x)^{-s-1}\,dx=\frac{\sqrt{\pi}}{(1+\sqrt{\beta})^{2s+1}\sqrt{\beta}}\frac{\Gamma\left(s+\frac 12\right)}{\Gamma(s+1)} \end{eqnarray*}$$
を示せばよい. これはGaussの超幾何積分になっているので,
$$\begin{eqnarray*} \int_0^1x^{s+1/2}(1-x)^{s-1/2}(1-(1-\beta)x)^{-s-1}\,dx=\frac{\Gamma\left(s+\frac 32\right)\Gamma\left(s+\frac 12\right)}{\Gamma(2s+2)}\,{}_2F_1\left[\begin{matrix}s+\frac 32,s+1\\2s+2\end{matrix};1-\beta\right] \end{eqnarray*}$$
ここで, Legendreの倍角公式より,
$$\begin{eqnarray*} \Gamma(2s+2)=\frac{2^{2s+1}}{\sqrt{\pi}}\Gamma(s+1)\Gamma\left(s+\frac 32\right) \end{eqnarray*}$$
また, 超幾何関数に関する等式,
$$\begin{eqnarray*} {}_2F_1\left[\begin{matrix}a,\frac 12+a\\2a\end{matrix};z\right]=\frac 1{\sqrt{1-z}}\left(\frac{2}{1+\sqrt{1-z}}\right)^{2a-1} \end{eqnarray*}$$
より,
$$\begin{eqnarray*} {}_2F_1\left[\begin{matrix}s+\frac 32,s+1\\2s+2\end{matrix};1-\beta\right]=\frac 1{\sqrt{\beta}}\left(\frac{2}{1+\sqrt{\beta}}\right)^{2s+1} \end{eqnarray*}$$
これらを合わせて,
$$\begin{eqnarray*} \int_0^1x^{s+1/2}(1-x)^{s-1/2}(1-(1-\beta)x)^{-s-1}\,dx=\frac{\sqrt{\pi}}{(1+\sqrt{\beta})^{2s+1}\sqrt{\beta}}\frac{\Gamma\left(s+\frac 12\right)}{\Gamma(s+1)} \end{eqnarray*}$$
を得る.

30個目( https://twitter.com/integralsbot/status/1361339989678649344 )

$$\begin{eqnarray*} \int_0^{\infty}\erfc\,x\,dx&=&\frac 1{\sqrt{\pi}}\\ \int_0^{\infty}\erfc^2\,x\,dx&=&\frac 2{\sqrt{\pi}}-\frac{\sqrt{2}}{\sqrt{\pi}}\\ \int_0^{\infty}\erfc^3\,x\,dx&=&\frac{3}{\sqrt{\pi}}-\frac{6\sqrt{2}}{\pi^{3/2}}\arctan\sqrt{2}\\ \int_0^{\infty}\erfc^4\,x\,dx&=&\frac{4}{\sqrt{\pi}}-\frac{24\sqrt{2}}{\pi^{3/2}}\arctan\frac 1{\sqrt{8}} \end{eqnarray*}$$

見慣れない関数なので, まず定義を確認します. $\erfc$は相補誤差関数といい,
$$\begin{eqnarray*} \erfc\,x:=\frac{2}{\sqrt{\pi}}\int_x^{\infty}e^{-z^2}\,dz \end{eqnarray*}$$
で定義されます.

まず, $0\leq n$に対し,
$$\begin{eqnarray*} I_n(a,b):=\int_0^{\infty}e^{-a^2x^2}\erfc^n\,bx\,dx \end{eqnarray*}$$
$I_{-1}(a,b):=0$とする.
定義より,
$$\begin{eqnarray*} I_0(a,b)&=&\int_0^{\infty}e^{-a^2x^2}\,dx\\ &=&\frac{\sqrt{\pi}}{2a} \end{eqnarray*}$$
である. また, $1\leq n$に対し,
$$\begin{eqnarray*} \frac{\partial}{\partial b}I_n(a,b)&=&-\frac{2n}{\sqrt{\pi}}\int_0^{\infty}xe^{-(a^2+b^2)x^2}\erfc^{n-1}\,bx\,dx\\ &=&-\frac{2n}{\sqrt{\pi}}\left(\left[-\frac {e^{-(a^2+b^2)x^2}}{2(a^2+b^2)}\erfc^{n-1}\,bx\right]_0^{\infty}-\frac{b(n-1)}{\sqrt{\pi}(a^2+b^2)}\int_0^{\infty}e^{-(a^2+2b^2)x^2}\erfc^{n-2}\,bx\,dx\right)\\ &=&-\frac{2n}{\sqrt{\pi}}\left(\frac 1{2(a^2+b^2)}-\frac{(n-1)b}{\sqrt{\pi}(a^2+b^2)}I_{n-2}(\sqrt{a^2+2b^2},b)\right)\\ &=&\frac{2n(n-1)b}{\pi(a^2+b^2)}I_{n-2}(\sqrt{a^2+2b^2},b)-\frac{n}{\sqrt{\pi}(a^2+b^2)} \end{eqnarray*}$$
これを用いて,
$$\begin{eqnarray*} \frac{\partial}{\partial b}I_1(a,b)=-\frac{1}{\sqrt{\pi}(a^2+b^2)} \end{eqnarray*}$$
より,
$$\begin{eqnarray*} I_1(a,b)=-\frac{1}{a\sqrt{\pi}}\arctan\frac ba+C_1(a) \end{eqnarray*}$$
$b=0$として, $C_1(a)=\frac{\sqrt{\pi}}{2a}$, 整理して,
$$\begin{eqnarray*} I_1(a,b)=\frac 1{a\sqrt{\pi}}\arctan\frac ab \end{eqnarray*}$$
また,
$$\begin{eqnarray*} \frac{\partial}{\partial b}I_2(a,b)&=&\frac{4b}{\pi(a^2+b^2)}\frac{\sqrt{\pi}}{2\sqrt{a^2+2b^2}}-\frac{2}{\sqrt{\pi}(a^2+b^2)}\\ &=&\frac{2}{\sqrt{\pi}}\left(\frac{b}{(a^2+b^2)\sqrt{a^2+2b^2}}-\frac 1{a^2+b^2}\right) \end{eqnarray*}$$
だから,
$$\begin{eqnarray*} I_2(a,b)=\frac 2{a\sqrt{\pi}}\left(\arctan\frac{\sqrt{a^2+2b^2}}{a}-\arctan\frac ba\right)+C_2(a) \end{eqnarray*}$$
$b=0$として, $C_2(a)=0$より,
$$\begin{eqnarray*} I_2(a,b)=\frac 2{a\sqrt{\pi}}\left(\arctan\frac{\sqrt{a^2+2b^2}}{a}-\arctan\frac ba\right) \end{eqnarray*}$$
これより,
$$\begin{eqnarray*} \int_0^{\infty}\erfc\,x\,dx&=&\lim_{a\to 0}I_1(a,1)=\frac 1{\sqrt{\pi}}\\ \int_0^{\infty}\erfc^2\,x\,dx&=&\lim_{a\to 0}I_2(a,1)=\frac{2-\sqrt{2}}{\sqrt{\pi}} \end{eqnarray*}$$
また, $2\leq n$に対し,
$$\begin{eqnarray*} \int_0^{\infty}\erfc^n\,x\,dx&=&\left[x\erfc^n\,x\right]_0^{\infty}+\frac{2n}{\sqrt{\pi}}\int_0^{\infty}xe^{-x^2}\erfc^{n-1}\,x\,dx\\ &=&\frac{n}{\sqrt{\pi}}\left[-e^{-x^2}\erfc^{n-1}\,x\right]_0^{\infty}-\frac{2n(n-1)}{\pi}\int_0^{\infty}e^{-2x^2}\erfc^{n-2}\,x\,dx\\ &=&\frac{n}{\sqrt{\pi}}-\frac{2n(n-1)}{\pi}I_{n-2}(\sqrt{2},1) \end{eqnarray*}$$
これより,
$$\begin{eqnarray*} \int_0^{\infty}\erfc^3\,x\,dx&=&\frac{3}{\sqrt{\pi}}-\frac{6\sqrt{2}}{\pi^{3/2}}\arctan\sqrt{2}\\ \int_0^{\infty}\erfc^4\,x\,dx&=&\frac{4}{\sqrt{\pi}}-\frac{24\sqrt{2}}{\pi^{3/2}}\arctan \frac 1{\sqrt{8}} \end{eqnarray*}$$

今回もなかなか大変でしたが, 8個示せたので, これで示せたのは24個ですね. 100個めざしていきたいと思います.

参考文献

[1]
Jonathan M. Borwein, Marc Chamberland, Integer Power of Arcsin, International Journal of Mathematics and Mathematical Sciences
投稿日:202138

この記事を高評価した人

高評価したユーザはいません

この記事に送られたバッジ

バッジはありません。

投稿者

Wataru
Wataru
404
25143
超幾何関数, 直交関数, 多重ゼータ値などに興味があります

コメント

他の人のコメント

コメントはありません。
読み込み中...
読み込み中